Ich habe Probleme mit der Beschleunigung in Polarkoordinaten

Also habe ich eine Frage zur Beschleunigung in Polarkoordinaten gelöst, aber die meisten Leute in meiner Klasse (Klassische Physik, erstes Jahr an der Universität, die Physik studieren) sind mit meiner Antwort nicht einverstanden. Die Frage bezieht sich also auf einen Spielplatzkarussell mit einem Radius von 3 m, der sich mit einer Geschwindigkeit von 10 m / s dreht. Eine Person im Kreisverkehr wirft einen Schneeball mit einer Geschwindigkeit von 20 m/s in Richtung Kreisverkehrmitte. Welche Beschleunigung wird der Schneeball erfahren, die dazu führen wird, dass er die Mitte verfehlt?

Also begann ich mit der Beschleunigungsformel in Polarkoordinaten:

A = R ^ ( R ¨ R θ ˙ 2 ) + θ ^ ( R θ ¨ + 2 R ˙ θ ˙ ) .

Ich habe die Mitte des Kreisverkehrs als Ursprung genommen und das Problem so interpretiert: R ist die Entfernung vom Schneeball zum Zentrum des Kreisverkehrs, R ˙ ist dann die eindimensionale Geschwindigkeit des Schneeballs, und R ¨ wäre die Änderungsrate von R ˙ . θ ˙ ist die Winkelgeschwindigkeit des Kreisverkehrs, und θ ¨ die Winkelbeschleunigung.

Aus der Problemstellung habe ich also die folgenden Werte herausgesucht:

R ¨ = 0 , R ˙ = 20  MS 1 , R = ( 3 20 T ) M , θ ¨ = 0 , θ ˙ = 3.33  rad s 1

Und dann habe ich diese Werte einfach in die obige Formel eingesetzt. Dann bekomme ich eine Antwort wo die Komponente aus A im R ^ Richtung hängt ab T , und es gibt eine konstante Komponente in der θ ^ Richtung.

Die meisten Leute in meiner Klasse haben diese Formel jedoch nicht verwendet und argumentieren, dass es nur eine Corilios-Beschleunigung geben sollte, in diesem Fall zB die Theta-Komponente, as θ ¨ = 0. Das macht für mich keinen Sinn, da ich den Eindruck hatte, dass die Beschleunigungsformel immer in Polarkoordinaten funktionieren sollte. So sieht es jedenfalls für mich aus, wenn die erste Zeile in der Ableitung so allgemein wie gerecht ist R = R R ^ , und dann nehmen wir die erste und zweite Ableitung von R

Also, was ich wissen will, ist, wer Recht hat? Habe ich Recht, dass wir diese Formel einfach anwenden können, oder haben meine Klassenkameraden Recht, und es gibt nur eine Corilios-Beschleunigung? Auch die Werte, die ich ausgewählt habe R ¨ , R ˙ , R , θ ¨ , θ ˙ richtig?

Jede Hilfe wäre sehr willkommen!

Du denkst das R ˙ ist die Geschwindigkeit des Schneeballs und θ ˙ ist die Winkelgeschwindigkeit des Kreisverkehrs. Warum denkst du, dass in einer Beschleunigungsformel R Und θ wären Koordinaten verschiedener Objekte?
Ich dachte, da der Schneeball das Ding ist, das eine Beschleunigung erfährt, sollten wir das sagen R ˙ ist die Geschwindigkeit des Schneeballs, und θ ˙ ist die Winkelgeschwindigkeit des Schneeballs. Entschuldigung, ich hätte deutlicher machen sollen, dass ich davon ausgegangen bin, dass der Schneeball und der Kreisverkehr dieselbe Winkelgeschwindigkeit haben würden, wie der Schneeball von einer Person auf den Kreisverkehr geworfen wird. Ist diese Annahme nicht richtig?
Wenn sie die gleiche Winkelgeschwindigkeit hätten, warum würde der Schneeball das Zentrum verfehlen?
Oh richtig, ja, ich denke, das würde es nicht. Das war dumm von mir. Entschuldigung, meine Intuition ist nicht die beste. Jetzt denke ich, dass die Winkelgeschwindigkeit des Schneeballs 0 sein sollte, aber wenn ich mir die Formel ansehe, scheint das nicht richtig zu sein, da dann jeder Term auf 0 geht. Können Sie mir einen Hinweis geben, wie ich die Winkelgeschwindigkeit des Schneeballs berechnen kann?
Das kannst du nicht vermuten R = R 0 R ˙ T . Ich verstehe, was Sie dachten, aber dies würde nur für einen intertialen Referenzrahmen gelten. Was Sie tun müssen, ist, dass Sie die Differentialgleichung mit den richtigen Startbedingungen lösen müssen.
@tomtom1-4 danke für deinen Kommentar. Es tut mir leid, ich habe gerade erst angefangen, mit solchen Dingen zu arbeiten. Ich bin mir nicht sicher, welche Differentialgleichung du meinst. Können Sie das näher erläutern?
Nun, es gibt keine externe Kraft. Dafür A = 0 . Und dann landen Sie bei einer gewöhnlichen Differentialgleichung.
Aber wenn ich es mir ansehe. Es scheint, als ob Sie es nicht rechtzeitig lösen können R ( T ) analytisch. Versuchen Sie, es in Bezug auf zu lösen θ .
Natürlich wäre es viel einfacher, wenn Sie es aus einem Trägheitsbezugssystem lösen und dann in den Kreisverkehr verwandeln würden.
Welche Beschleunigung wird der Schneeball erfahren, die dazu führen wird, dass er die Mitte verfehlt? Du meinst, während es geworfen wird? Nachdem es geworfen wurde, wird es nur durch die Schwerkraft beschleunigt.
Ich denke, wir können davon ausgehen, dass die Geschwindigkeit von 10 m/s am Rand bei R = 3 m liegt, aber ab welchem ​​R wird der Schneeball geworfen?
Ich stimme TomTom zu. Im Trägheitsrahmen bewegt sich der Schneeball mit konstanter Geschwindigkeit mit radialen und tangentialen Komponenten.

Antworten (4)

Ignorieren der z-Bewegung im Folgenden.

Referenzrahmen: „Labor“ – der, in dem sich der Kreisverkehr dreht. Rechtshänder, Ausgangspunkt Kreisverkehr Mitte.

Die Flugbahn ist eine gerade Linie. Es gibt keine Beschleunigung. Der Grund, warum der Ball die Mitte verfehlt, liegt darin, dass seine Anfangsbedingungen so sind - es gab immer eine anfängliche Tangente ( θ ^ ) Geschwindigkeit.

Bezugsrahmen: „rotierend“ – der, in dem der Kreisverkehr ruht. Fällt mit Labor bei zusammen T = 0

Bei T = 0
Das Objekt hat nur Radialgeschwindigkeit ( R ^ ). Theoretisch sollte es die Mitte treffen. Der einzige Grund, warum es nicht wird, ist, wenn etwas es tangential beschleunigt. Diese kommen von den Pseudokräften. Das Objekt erfährt eine Beschleunigung:

  1. Koriolis: ω × v . Hier seit v ^ = R ^ , die Beschleunigung ist genau das, was wir wollen: entlang θ ^ .
  2. Zentrifugal: ω × ( ω × R ) . Hier seit v ^ = R ^ , die Beschleunigung ist mit R ^ . Wirkt sich nicht auf das Schlagen der Mitte aus.

Bei T > 0

  1. Das Objekt beginnt sich tangential zu bewegen. Gleichzeitig wird seine Radialgeschwindigkeit durch die Zentrifugalkraft verringert. Auch die Coriolis-Kraft aus tangentialer Bewegung ist ebenfalls entlang der Zentrifugalkraft. Alles in allem bewegt sich das Objekt, als würde es vorwärts gehen, während es sich in Rotationsrichtung krümmt (siehe Abb. 1 unten).
  2. Schließlich dreht das Objekt sofort um und scheint aus dem Kreisverkehr zu entkommen (siehe Abb. 2 unten).
  3. jetzt wechselt die Richtung der Coriolis-Kraft....

Insgesamt bewegt sich das Objekt in einer immer größer werdenden Spirale. Beachten Sie, dass sich die Beschleunigung mit der Zeit ändert.

Abschluss

Wessen Rahmen sollten wir also berücksichtigen? Hängt vom Betrachter ab – wenn es die Person auf dem Kreisverkehr ist, ist es der rotierende Rahmen. Die Endbeschleunigung muss natürlich die Schwerkraft beinhalten. Die angegebenen Werte für R ¨ , R ˙ , R , θ ¨ , θ ˙ scheinen für den Laborrahmen richtig zu sein.

Im Drehrahmen
R ¨ 0 , R ˙ = 20  MS 1 , R ( 3 20 T ) M , θ ¨ 0 , θ ˙ = 0  rad s 1


Abb. 1: Flugbahn zu einem bestimmten Zeitpunkt
Abb. 2: Flugbahn anfänglich. (Die blaue Kurve ist die Trajektorie, die im rotierenden Rahmen zu sehen ist. Orange ist die Stelle, an der sich die Person im Laborrahmen befinden würde. Die X- und Y-Achsen sind die X- und Y-Positionen in Metern.)

Abb. 2: Flugbahn zu einem späteren Zeitpunkt
Abb. 2: Flugbahn nach einiger Zeit. (Die blaue Kurve ist die Trajektorie, die im rotierenden Rahmen zu sehen ist. Orange ist dort, wo sich die Person im Laborrahmen befinden würde.)


Soweit die von Ihnen angegebene Formel gilt, gilt sie nur für Trägheitsrahmen. Verwenden Sie insbesondere für Drehrahmen ( ' bezeichnet rotierenden Rahmen)

M A ' = F ' M D ω D T × R ' 2 M ω × M v ' M ω × ( ω × R ' )
mit
F ' = R ^ ' ( R ¨ ' R ' θ ˙ ' 2 ) + θ ^ ' ( R ' θ ¨ ' + 2 R ˙ ' θ ˙ ' ) v ' = R ˙ ' R ^ ' + R ' θ ˙ ' θ ^ '

Bei T = 0 , ohne angewandte Kraft
ω = ω z ^ ' R ' = R R ^ ' v = v R ^ ' F ' = 0  
wir bekommen
A ' = 2 M ω v θ ^ ' + M ω 2 R R ^ '

Sehr gute Antwort! Ihre Gleichung für F' muss die Masse enthalten.

Vielleicht macht das den Sinn der Übung zunichte, aber wenn Sie wissen wollen, was der Schneeball macht, warum berechnen Sie dann nicht einfach die Bewegung des Schneeballs, die überhaupt nichts mit der Bewegung des Kreisels zu tun hat? Es fliegt einfach in einer Parabel, deren horizontaler Teil eine gerade Linie relativ zum Boden ist. Wenn die Anfangsgeschwindigkeit in Richtung der Mitte des Kreisverkehrs verläuft, trifft sie auf die Mitte des Kreisverkehrs.

Auch wenn Sie eine Berechnung in einem anderen Rahmen durchführen möchten, ist es sicherlich hilfreich zu wissen, wie die Antwort lautet, wenn Sie sie am einfachsten berechnen.

Ich denke, das Problem bedeutet, dass die anfängliche Vektorgeschwindigkeit nicht in Richtung Zentrum geht; es ist die Geschwindigkeitskomponente vom Wurf zur Mitte hin plus die Geschwindigkeitskomponente aufgrund der Drehung des Karussells.

Die radiale Anfangsgeschwindigkeit ist die Ursache für das Ausscheren in einem rotierenden Rahmen.

Im dynamischen Umfangsgleichgewicht gleicht die Coreolis-Beschleunigung die Winkelbeschleunigung aus

a R = 2 ω R ˙ R A D ich A l
und ist die De-Tracking-Komponente.

Wenn die Schwerkraft vorhanden ist, fügt sie eine Verschiebung in vertikaler Richtung hinzu, beeinflusst jedoch nicht die Verschiebung in der Ebene des rotierenden Karussells. Daher werde ich die Wirkung der Schwerkraft ignorieren.

Im Trägheitsrahmen gibt es keine Kraft, also keine Beschleunigung, aber der Ball hat sowohl eine anfängliche tangentiale als auch radiale Geschwindigkeit. Tangential von der Bewegung des rotierenden Karussells und radial von der Person, die den Ball wirft. Der Ball verfehlt das Zentrum, da die Anfangsgeschwindigkeit nicht rein radial ist. Ihre Beziehung für die Beschleunigung A in Polarkoordinaten liegt im Inertialsystem und in diesem System A = 0 da es keine Nettokraft im Trägheitsrahmen gibt. Da keine Kraft vorhanden ist, bewegt sich der Ball mit konstanter Geschwindigkeit in einer geraden Linie, verfehlt jedoch das Zentrum.

In dem sich mit dem Karussell drehenden Nichtträgheitsrahmen ist die Anfangsgeschwindigkeit zum Zentrum hin gerichtet. Aber die Beschleunigung in diesem Rahmen ist aufgrund von fiktiven Kräften nicht Null, also ist die Geschwindigkeit in diesem Rahmen nicht konstant. @lineage gibt Ihnen die Bewegungsgleichung im nicht trägen rotierenden Rahmen als M A ' . Sein F ' ist die Gesamtkraft im Trägheitsrahmen und ist Null, wie oben diskutiert; seine anderen Begriffe für M A ' umfassen alle fiktiven Rotationskräfte im Allgemeinen. Für dieses Problem ω Konstant ist also die fiktive Kraft M D ω D T × R ' , manchmal Euler-Kraft genannt, ist Null. Die verbleibenden zwei fiktiven Kräfte M ω × ( ω × R ' ) , die Zentrifugalkraft und 2 M ω × D R ' D T , die Coriolis-Kraft, beeinflussen die Bewegung, wie sie im rotierenden Rahmen betrachtet wird. Die Zentrifugalkraft wirkt radial nach außen und hindert die Kugel nicht daran, sich zur Mitte zu bewegen. Die Coriolis-Kraft ist die fiktive Kraft im rotierenden, nicht trägen Rahmen, die verhindert, dass sich die Kugel in die Mitte bewegt.

Die Position des Teilchens als Funktion der Zeit kann erzeugt werden, indem die Bewegungsgleichung entweder im inertialen oder im nicht-inertialen Rotationssystem gelöst wird. Für dieses Problem ist die Bewegung im Trägheitsrahmen unter Verwendung kartesischer Koordinaten eine gerade Linie, da keine Kraft vorhanden ist, und die Bewegung unter Verwendung von Polarkoordinaten sowohl im Trägheits- als auch im rotierenden Rahmen kann unter Verwendung von Trigonometrie entwickelt werden.

In Abbildung 1 ist P(t) die Position des Balls zum Zeitpunkt t und wird durch drei verschiedene Koordinatensätze ausgedrückt: kartesisch ( X , j ) im Inertialsystem, polar ( R , θ ) im Inertialsystem und polar ( R , β ) im nicht trägen Drehrahmen. Der rotierende Rahmen ist rot dargestellt und hat einen Winkel a ( T ) in Bezug auf den Trägheitsrahmen; D a ( T ) D T = ω ist konstant. P(t) in allen drei Koordinaten wird in Abbildung 1 ausgewertet.

Abbildung 2 zeigt die Bewegung im kartesischen Inertialsystem. Fig. 3 die Bewegung unter Verwendung von Polarkoordinaten sowohl im Trägheits- als auch im Rotationsrahmen zeigt; R ( T ) ist in beiden Rahmen gleich.

P(t)

Trägheit kartesisch

Geben Sie hier die Bildbeschreibung ein